Why is the Ratio of $ln(x)$ and $log(x)$ a constant?












10












$begingroup$


I was solving some "Big-Oh" algorithm asymptotic complexity problems, when I discovered that for some constant $c$ and some variable $x$:



$$c^{log(x)}$$ and $$x^{log(c)}$$



grow at the same rate. When figuring this out I ended up with the expression:



$$frac{ln(x)}{log(x)}=frac{ln(c)}{log(c)}approx 2.3025$$



This result was surpirsing and somewhat baffling to me. This might seem like a naive question, but could someone help me understand how the ratio of $log_{10}(x)$ and $ln(x)$ ends up being a constant value?










share|cite|improve this question











$endgroup$








  • 1




    $begingroup$
    Indeed the expressions you gave are equal, not just of equal growth: $$c^{log(x)}=e^{log(c)log(x)}=x^{log(c )}$$
    $endgroup$
    – adfriedman
    Jan 27 at 7:00












  • $begingroup$
    This is just the change of base formula for logarithms that you learn in basic algebra.
    $endgroup$
    – Brady Gilg
    Jan 29 at 23:33


















10












$begingroup$


I was solving some "Big-Oh" algorithm asymptotic complexity problems, when I discovered that for some constant $c$ and some variable $x$:



$$c^{log(x)}$$ and $$x^{log(c)}$$



grow at the same rate. When figuring this out I ended up with the expression:



$$frac{ln(x)}{log(x)}=frac{ln(c)}{log(c)}approx 2.3025$$



This result was surpirsing and somewhat baffling to me. This might seem like a naive question, but could someone help me understand how the ratio of $log_{10}(x)$ and $ln(x)$ ends up being a constant value?










share|cite|improve this question











$endgroup$








  • 1




    $begingroup$
    Indeed the expressions you gave are equal, not just of equal growth: $$c^{log(x)}=e^{log(c)log(x)}=x^{log(c )}$$
    $endgroup$
    – adfriedman
    Jan 27 at 7:00












  • $begingroup$
    This is just the change of base formula for logarithms that you learn in basic algebra.
    $endgroup$
    – Brady Gilg
    Jan 29 at 23:33
















10












10








10


3



$begingroup$


I was solving some "Big-Oh" algorithm asymptotic complexity problems, when I discovered that for some constant $c$ and some variable $x$:



$$c^{log(x)}$$ and $$x^{log(c)}$$



grow at the same rate. When figuring this out I ended up with the expression:



$$frac{ln(x)}{log(x)}=frac{ln(c)}{log(c)}approx 2.3025$$



This result was surpirsing and somewhat baffling to me. This might seem like a naive question, but could someone help me understand how the ratio of $log_{10}(x)$ and $ln(x)$ ends up being a constant value?










share|cite|improve this question











$endgroup$




I was solving some "Big-Oh" algorithm asymptotic complexity problems, when I discovered that for some constant $c$ and some variable $x$:



$$c^{log(x)}$$ and $$x^{log(c)}$$



grow at the same rate. When figuring this out I ended up with the expression:



$$frac{ln(x)}{log(x)}=frac{ln(c)}{log(c)}approx 2.3025$$



This result was surpirsing and somewhat baffling to me. This might seem like a naive question, but could someone help me understand how the ratio of $log_{10}(x)$ and $ln(x)$ ends up being a constant value?







logarithms






share|cite|improve this question















share|cite|improve this question













share|cite|improve this question




share|cite|improve this question








edited Jan 28 at 8:52









Asaf Karagila

305k32435765




305k32435765










asked Jan 27 at 6:41









user3776749user3776749

332212




332212








  • 1




    $begingroup$
    Indeed the expressions you gave are equal, not just of equal growth: $$c^{log(x)}=e^{log(c)log(x)}=x^{log(c )}$$
    $endgroup$
    – adfriedman
    Jan 27 at 7:00












  • $begingroup$
    This is just the change of base formula for logarithms that you learn in basic algebra.
    $endgroup$
    – Brady Gilg
    Jan 29 at 23:33
















  • 1




    $begingroup$
    Indeed the expressions you gave are equal, not just of equal growth: $$c^{log(x)}=e^{log(c)log(x)}=x^{log(c )}$$
    $endgroup$
    – adfriedman
    Jan 27 at 7:00












  • $begingroup$
    This is just the change of base formula for logarithms that you learn in basic algebra.
    $endgroup$
    – Brady Gilg
    Jan 29 at 23:33










1




1




$begingroup$
Indeed the expressions you gave are equal, not just of equal growth: $$c^{log(x)}=e^{log(c)log(x)}=x^{log(c )}$$
$endgroup$
– adfriedman
Jan 27 at 7:00






$begingroup$
Indeed the expressions you gave are equal, not just of equal growth: $$c^{log(x)}=e^{log(c)log(x)}=x^{log(c )}$$
$endgroup$
– adfriedman
Jan 27 at 7:00














$begingroup$
This is just the change of base formula for logarithms that you learn in basic algebra.
$endgroup$
– Brady Gilg
Jan 29 at 23:33






$begingroup$
This is just the change of base formula for logarithms that you learn in basic algebra.
$endgroup$
– Brady Gilg
Jan 29 at 23:33












7 Answers
7






active

oldest

votes


















15












$begingroup$

For all $xneq1$, $x>0$ we have: $$frac{ln{x}}{log{x}}=frac{ln{x}}{frac{log_ex}{log_e{10}}}=log_e10=ln10.$$






share|cite|improve this answer











$endgroup$





















    37












    $begingroup$

    Some of the answers already provided get close to a full explanation, but not quite.



    Recall that if $b > 1$ and $x > 0$, and $$log_b x = y,$$ what this means is that $b^y = x$. In other words, the base-$b$ logarithm of $x$ is an exponent $y$ such that when the base $b$ is raised to the $y^{rm th}$ power, the result is $x$. This is the definition of a (real-valued) logarithm.



    So, why is it that for two bases $a$, $b$, $$frac{log_a x}{log_b x}$$ is a constant not dependent on $x$? The reason is that $log_a x$ is an exponent, say $y$, such that $a^y = x$; and $log_b x$ is an exponent, say $w$, such that $b^w = x$; then $$b^w = x = a^y.$$ And now, raising both sides to the $1/w$ power, we get $$b = (b^w)^{1/w} = (a^y)^{1/w} = a^{y/w}.$$ So the ratio $y/w$ does not depend on $x$. In fact, again using the definition of logarithm, $y/w$ is the exponent for which the base $a$ must be raised to yield $b$; that is, we explicitly have $$frac{y}{w} = log_a b,$$ and from this, we get (with one additional algebraic step) what is known as the "change-of-base" formula $$frac{log_a x}{log_a b} = log_b x.$$



    Note that only the definition of $log$ was used, and the rule for exponents $(b^m)^n = b^{mn}$.






    share|cite|improve this answer









    $endgroup$





















      10












      $begingroup$

      Hint: $ln(10)approx 2.3025$. Given that information, your conjecture is that $ln(x)=ln(10)log_{10}(x)$. Can you see a way to prove that?






      share|cite|improve this answer









      $endgroup$













      • $begingroup$
        I love the effort of guiding people to the answer.
        $endgroup$
        – Git Gud
        Jan 27 at 14:25



















      3












      $begingroup$

      If $ln x = a_x$ and $log_{10} x = b_x$ then



      $e^{a_x} = x$ and $10^{b_x} = x$.



      Bear in mind $10 = e^{ln 10}$ so $10^k = (e^{ln 10})^k = e^{kln 10}$.



      So if $10^{k_x} =e^{k_xln 10} = x$ then......



      By definition $log_{10} x = k_x$ and $ln x = k_xln 10$ and so.......



      $frac {ln x}{log_{10} x} = frac {k_xln 10}{k_x} = ln 10$.



      It's just a conversion constant and shouldn't surprise us.



      This is the very basis of the rule $log_b x = frac {log_a x}{log_a b}$ (note if $x$ is a variable and $b$ is a constant that is exactly your observation).






      share|cite|improve this answer









      $endgroup$





















        3












        $begingroup$

        Take the logarithm of both expressions:
        $$
        log(c^{log x})=log xlog c
        qquad
        log(x^{log c})=log clog x
        $$

        So, not only $c^{log x}$ and $x^{log c}$ grow at the same rate: they're equal, whatever base of logarithms you use.



        For the second part, note that $x=e^{ln x}=b^{log_bx}$ by definition. Then
        $$
        ln x=log_bxln b
        $$

        Therefore, for $xne1$,
        $$
        frac{ln x}{log_bx}=ln b
        $$






        share|cite|improve this answer









        $endgroup$





















          1












          $begingroup$

          $log_a b$ = $log_c b over log_c a$ is a general rule.
          Thus $log_c a = {log_c b over log_a b}$ and your expression is the case of this when $a=10, b=x, c=e$.






          share|cite|improve this answer











          $endgroup$













          • $begingroup$
            Ummm.... by your correct rule we have $$ln(10)=frac{log_x(10)}{log_x(e)}$$ which isn't what we're trying to prove.
            $endgroup$
            – Rhys Hughes
            Jan 27 at 6:54








          • 1




            $begingroup$
            Thanks. I edited to correct
            $endgroup$
            – J. W. Tanner
            Jan 27 at 7:10



















          1












          $begingroup$

          Because $e^{cx}=10^{x}$ for all $x$ and $c:=ln(10)approx 2.3025$






          share|cite|improve this answer











          $endgroup$













            Your Answer





            StackExchange.ifUsing("editor", function () {
            return StackExchange.using("mathjaxEditing", function () {
            StackExchange.MarkdownEditor.creationCallbacks.add(function (editor, postfix) {
            StackExchange.mathjaxEditing.prepareWmdForMathJax(editor, postfix, [["$", "$"], ["\\(","\\)"]]);
            });
            });
            }, "mathjax-editing");

            StackExchange.ready(function() {
            var channelOptions = {
            tags: "".split(" "),
            id: "69"
            };
            initTagRenderer("".split(" "), "".split(" "), channelOptions);

            StackExchange.using("externalEditor", function() {
            // Have to fire editor after snippets, if snippets enabled
            if (StackExchange.settings.snippets.snippetsEnabled) {
            StackExchange.using("snippets", function() {
            createEditor();
            });
            }
            else {
            createEditor();
            }
            });

            function createEditor() {
            StackExchange.prepareEditor({
            heartbeatType: 'answer',
            autoActivateHeartbeat: false,
            convertImagesToLinks: true,
            noModals: true,
            showLowRepImageUploadWarning: true,
            reputationToPostImages: 10,
            bindNavPrevention: true,
            postfix: "",
            imageUploader: {
            brandingHtml: "Powered by u003ca class="icon-imgur-white" href="https://imgur.com/"u003eu003c/au003e",
            contentPolicyHtml: "User contributions licensed under u003ca href="https://creativecommons.org/licenses/by-sa/3.0/"u003ecc by-sa 3.0 with attribution requiredu003c/au003e u003ca href="https://stackoverflow.com/legal/content-policy"u003e(content policy)u003c/au003e",
            allowUrls: true
            },
            noCode: true, onDemand: true,
            discardSelector: ".discard-answer"
            ,immediatelyShowMarkdownHelp:true
            });


            }
            });














            draft saved

            draft discarded


















            StackExchange.ready(
            function () {
            StackExchange.openid.initPostLogin('.new-post-login', 'https%3a%2f%2fmath.stackexchange.com%2fquestions%2f3089210%2fwhy-is-the-ratio-of-lnx-and-logx-a-constant%23new-answer', 'question_page');
            }
            );

            Post as a guest















            Required, but never shown

























            7 Answers
            7






            active

            oldest

            votes








            7 Answers
            7






            active

            oldest

            votes









            active

            oldest

            votes






            active

            oldest

            votes









            15












            $begingroup$

            For all $xneq1$, $x>0$ we have: $$frac{ln{x}}{log{x}}=frac{ln{x}}{frac{log_ex}{log_e{10}}}=log_e10=ln10.$$






            share|cite|improve this answer











            $endgroup$


















              15












              $begingroup$

              For all $xneq1$, $x>0$ we have: $$frac{ln{x}}{log{x}}=frac{ln{x}}{frac{log_ex}{log_e{10}}}=log_e10=ln10.$$






              share|cite|improve this answer











              $endgroup$
















                15












                15








                15





                $begingroup$

                For all $xneq1$, $x>0$ we have: $$frac{ln{x}}{log{x}}=frac{ln{x}}{frac{log_ex}{log_e{10}}}=log_e10=ln10.$$






                share|cite|improve this answer











                $endgroup$



                For all $xneq1$, $x>0$ we have: $$frac{ln{x}}{log{x}}=frac{ln{x}}{frac{log_ex}{log_e{10}}}=log_e10=ln10.$$







                share|cite|improve this answer














                share|cite|improve this answer



                share|cite|improve this answer








                edited Jan 27 at 7:55

























                answered Jan 27 at 6:57









                Michael RozenbergMichael Rozenberg

                104k1891196




                104k1891196























                    37












                    $begingroup$

                    Some of the answers already provided get close to a full explanation, but not quite.



                    Recall that if $b > 1$ and $x > 0$, and $$log_b x = y,$$ what this means is that $b^y = x$. In other words, the base-$b$ logarithm of $x$ is an exponent $y$ such that when the base $b$ is raised to the $y^{rm th}$ power, the result is $x$. This is the definition of a (real-valued) logarithm.



                    So, why is it that for two bases $a$, $b$, $$frac{log_a x}{log_b x}$$ is a constant not dependent on $x$? The reason is that $log_a x$ is an exponent, say $y$, such that $a^y = x$; and $log_b x$ is an exponent, say $w$, such that $b^w = x$; then $$b^w = x = a^y.$$ And now, raising both sides to the $1/w$ power, we get $$b = (b^w)^{1/w} = (a^y)^{1/w} = a^{y/w}.$$ So the ratio $y/w$ does not depend on $x$. In fact, again using the definition of logarithm, $y/w$ is the exponent for which the base $a$ must be raised to yield $b$; that is, we explicitly have $$frac{y}{w} = log_a b,$$ and from this, we get (with one additional algebraic step) what is known as the "change-of-base" formula $$frac{log_a x}{log_a b} = log_b x.$$



                    Note that only the definition of $log$ was used, and the rule for exponents $(b^m)^n = b^{mn}$.






                    share|cite|improve this answer









                    $endgroup$


















                      37












                      $begingroup$

                      Some of the answers already provided get close to a full explanation, but not quite.



                      Recall that if $b > 1$ and $x > 0$, and $$log_b x = y,$$ what this means is that $b^y = x$. In other words, the base-$b$ logarithm of $x$ is an exponent $y$ such that when the base $b$ is raised to the $y^{rm th}$ power, the result is $x$. This is the definition of a (real-valued) logarithm.



                      So, why is it that for two bases $a$, $b$, $$frac{log_a x}{log_b x}$$ is a constant not dependent on $x$? The reason is that $log_a x$ is an exponent, say $y$, such that $a^y = x$; and $log_b x$ is an exponent, say $w$, such that $b^w = x$; then $$b^w = x = a^y.$$ And now, raising both sides to the $1/w$ power, we get $$b = (b^w)^{1/w} = (a^y)^{1/w} = a^{y/w}.$$ So the ratio $y/w$ does not depend on $x$. In fact, again using the definition of logarithm, $y/w$ is the exponent for which the base $a$ must be raised to yield $b$; that is, we explicitly have $$frac{y}{w} = log_a b,$$ and from this, we get (with one additional algebraic step) what is known as the "change-of-base" formula $$frac{log_a x}{log_a b} = log_b x.$$



                      Note that only the definition of $log$ was used, and the rule for exponents $(b^m)^n = b^{mn}$.






                      share|cite|improve this answer









                      $endgroup$
















                        37












                        37








                        37





                        $begingroup$

                        Some of the answers already provided get close to a full explanation, but not quite.



                        Recall that if $b > 1$ and $x > 0$, and $$log_b x = y,$$ what this means is that $b^y = x$. In other words, the base-$b$ logarithm of $x$ is an exponent $y$ such that when the base $b$ is raised to the $y^{rm th}$ power, the result is $x$. This is the definition of a (real-valued) logarithm.



                        So, why is it that for two bases $a$, $b$, $$frac{log_a x}{log_b x}$$ is a constant not dependent on $x$? The reason is that $log_a x$ is an exponent, say $y$, such that $a^y = x$; and $log_b x$ is an exponent, say $w$, such that $b^w = x$; then $$b^w = x = a^y.$$ And now, raising both sides to the $1/w$ power, we get $$b = (b^w)^{1/w} = (a^y)^{1/w} = a^{y/w}.$$ So the ratio $y/w$ does not depend on $x$. In fact, again using the definition of logarithm, $y/w$ is the exponent for which the base $a$ must be raised to yield $b$; that is, we explicitly have $$frac{y}{w} = log_a b,$$ and from this, we get (with one additional algebraic step) what is known as the "change-of-base" formula $$frac{log_a x}{log_a b} = log_b x.$$



                        Note that only the definition of $log$ was used, and the rule for exponents $(b^m)^n = b^{mn}$.






                        share|cite|improve this answer









                        $endgroup$



                        Some of the answers already provided get close to a full explanation, but not quite.



                        Recall that if $b > 1$ and $x > 0$, and $$log_b x = y,$$ what this means is that $b^y = x$. In other words, the base-$b$ logarithm of $x$ is an exponent $y$ such that when the base $b$ is raised to the $y^{rm th}$ power, the result is $x$. This is the definition of a (real-valued) logarithm.



                        So, why is it that for two bases $a$, $b$, $$frac{log_a x}{log_b x}$$ is a constant not dependent on $x$? The reason is that $log_a x$ is an exponent, say $y$, such that $a^y = x$; and $log_b x$ is an exponent, say $w$, such that $b^w = x$; then $$b^w = x = a^y.$$ And now, raising both sides to the $1/w$ power, we get $$b = (b^w)^{1/w} = (a^y)^{1/w} = a^{y/w}.$$ So the ratio $y/w$ does not depend on $x$. In fact, again using the definition of logarithm, $y/w$ is the exponent for which the base $a$ must be raised to yield $b$; that is, we explicitly have $$frac{y}{w} = log_a b,$$ and from this, we get (with one additional algebraic step) what is known as the "change-of-base" formula $$frac{log_a x}{log_a b} = log_b x.$$



                        Note that only the definition of $log$ was used, and the rule for exponents $(b^m)^n = b^{mn}$.







                        share|cite|improve this answer












                        share|cite|improve this answer



                        share|cite|improve this answer










                        answered Jan 27 at 8:03









                        heropupheropup

                        63.8k762102




                        63.8k762102























                            10












                            $begingroup$

                            Hint: $ln(10)approx 2.3025$. Given that information, your conjecture is that $ln(x)=ln(10)log_{10}(x)$. Can you see a way to prove that?






                            share|cite|improve this answer









                            $endgroup$













                            • $begingroup$
                              I love the effort of guiding people to the answer.
                              $endgroup$
                              – Git Gud
                              Jan 27 at 14:25
















                            10












                            $begingroup$

                            Hint: $ln(10)approx 2.3025$. Given that information, your conjecture is that $ln(x)=ln(10)log_{10}(x)$. Can you see a way to prove that?






                            share|cite|improve this answer









                            $endgroup$













                            • $begingroup$
                              I love the effort of guiding people to the answer.
                              $endgroup$
                              – Git Gud
                              Jan 27 at 14:25














                            10












                            10








                            10





                            $begingroup$

                            Hint: $ln(10)approx 2.3025$. Given that information, your conjecture is that $ln(x)=ln(10)log_{10}(x)$. Can you see a way to prove that?






                            share|cite|improve this answer









                            $endgroup$



                            Hint: $ln(10)approx 2.3025$. Given that information, your conjecture is that $ln(x)=ln(10)log_{10}(x)$. Can you see a way to prove that?







                            share|cite|improve this answer












                            share|cite|improve this answer



                            share|cite|improve this answer










                            answered Jan 27 at 6:49









                            Chris CulterChris Culter

                            21.3k43887




                            21.3k43887












                            • $begingroup$
                              I love the effort of guiding people to the answer.
                              $endgroup$
                              – Git Gud
                              Jan 27 at 14:25


















                            • $begingroup$
                              I love the effort of guiding people to the answer.
                              $endgroup$
                              – Git Gud
                              Jan 27 at 14:25
















                            $begingroup$
                            I love the effort of guiding people to the answer.
                            $endgroup$
                            – Git Gud
                            Jan 27 at 14:25




                            $begingroup$
                            I love the effort of guiding people to the answer.
                            $endgroup$
                            – Git Gud
                            Jan 27 at 14:25











                            3












                            $begingroup$

                            If $ln x = a_x$ and $log_{10} x = b_x$ then



                            $e^{a_x} = x$ and $10^{b_x} = x$.



                            Bear in mind $10 = e^{ln 10}$ so $10^k = (e^{ln 10})^k = e^{kln 10}$.



                            So if $10^{k_x} =e^{k_xln 10} = x$ then......



                            By definition $log_{10} x = k_x$ and $ln x = k_xln 10$ and so.......



                            $frac {ln x}{log_{10} x} = frac {k_xln 10}{k_x} = ln 10$.



                            It's just a conversion constant and shouldn't surprise us.



                            This is the very basis of the rule $log_b x = frac {log_a x}{log_a b}$ (note if $x$ is a variable and $b$ is a constant that is exactly your observation).






                            share|cite|improve this answer









                            $endgroup$


















                              3












                              $begingroup$

                              If $ln x = a_x$ and $log_{10} x = b_x$ then



                              $e^{a_x} = x$ and $10^{b_x} = x$.



                              Bear in mind $10 = e^{ln 10}$ so $10^k = (e^{ln 10})^k = e^{kln 10}$.



                              So if $10^{k_x} =e^{k_xln 10} = x$ then......



                              By definition $log_{10} x = k_x$ and $ln x = k_xln 10$ and so.......



                              $frac {ln x}{log_{10} x} = frac {k_xln 10}{k_x} = ln 10$.



                              It's just a conversion constant and shouldn't surprise us.



                              This is the very basis of the rule $log_b x = frac {log_a x}{log_a b}$ (note if $x$ is a variable and $b$ is a constant that is exactly your observation).






                              share|cite|improve this answer









                              $endgroup$
















                                3












                                3








                                3





                                $begingroup$

                                If $ln x = a_x$ and $log_{10} x = b_x$ then



                                $e^{a_x} = x$ and $10^{b_x} = x$.



                                Bear in mind $10 = e^{ln 10}$ so $10^k = (e^{ln 10})^k = e^{kln 10}$.



                                So if $10^{k_x} =e^{k_xln 10} = x$ then......



                                By definition $log_{10} x = k_x$ and $ln x = k_xln 10$ and so.......



                                $frac {ln x}{log_{10} x} = frac {k_xln 10}{k_x} = ln 10$.



                                It's just a conversion constant and shouldn't surprise us.



                                This is the very basis of the rule $log_b x = frac {log_a x}{log_a b}$ (note if $x$ is a variable and $b$ is a constant that is exactly your observation).






                                share|cite|improve this answer









                                $endgroup$



                                If $ln x = a_x$ and $log_{10} x = b_x$ then



                                $e^{a_x} = x$ and $10^{b_x} = x$.



                                Bear in mind $10 = e^{ln 10}$ so $10^k = (e^{ln 10})^k = e^{kln 10}$.



                                So if $10^{k_x} =e^{k_xln 10} = x$ then......



                                By definition $log_{10} x = k_x$ and $ln x = k_xln 10$ and so.......



                                $frac {ln x}{log_{10} x} = frac {k_xln 10}{k_x} = ln 10$.



                                It's just a conversion constant and shouldn't surprise us.



                                This is the very basis of the rule $log_b x = frac {log_a x}{log_a b}$ (note if $x$ is a variable and $b$ is a constant that is exactly your observation).







                                share|cite|improve this answer












                                share|cite|improve this answer



                                share|cite|improve this answer










                                answered Jan 27 at 7:17









                                fleabloodfleablood

                                71k22686




                                71k22686























                                    3












                                    $begingroup$

                                    Take the logarithm of both expressions:
                                    $$
                                    log(c^{log x})=log xlog c
                                    qquad
                                    log(x^{log c})=log clog x
                                    $$

                                    So, not only $c^{log x}$ and $x^{log c}$ grow at the same rate: they're equal, whatever base of logarithms you use.



                                    For the second part, note that $x=e^{ln x}=b^{log_bx}$ by definition. Then
                                    $$
                                    ln x=log_bxln b
                                    $$

                                    Therefore, for $xne1$,
                                    $$
                                    frac{ln x}{log_bx}=ln b
                                    $$






                                    share|cite|improve this answer









                                    $endgroup$


















                                      3












                                      $begingroup$

                                      Take the logarithm of both expressions:
                                      $$
                                      log(c^{log x})=log xlog c
                                      qquad
                                      log(x^{log c})=log clog x
                                      $$

                                      So, not only $c^{log x}$ and $x^{log c}$ grow at the same rate: they're equal, whatever base of logarithms you use.



                                      For the second part, note that $x=e^{ln x}=b^{log_bx}$ by definition. Then
                                      $$
                                      ln x=log_bxln b
                                      $$

                                      Therefore, for $xne1$,
                                      $$
                                      frac{ln x}{log_bx}=ln b
                                      $$






                                      share|cite|improve this answer









                                      $endgroup$
















                                        3












                                        3








                                        3





                                        $begingroup$

                                        Take the logarithm of both expressions:
                                        $$
                                        log(c^{log x})=log xlog c
                                        qquad
                                        log(x^{log c})=log clog x
                                        $$

                                        So, not only $c^{log x}$ and $x^{log c}$ grow at the same rate: they're equal, whatever base of logarithms you use.



                                        For the second part, note that $x=e^{ln x}=b^{log_bx}$ by definition. Then
                                        $$
                                        ln x=log_bxln b
                                        $$

                                        Therefore, for $xne1$,
                                        $$
                                        frac{ln x}{log_bx}=ln b
                                        $$






                                        share|cite|improve this answer









                                        $endgroup$



                                        Take the logarithm of both expressions:
                                        $$
                                        log(c^{log x})=log xlog c
                                        qquad
                                        log(x^{log c})=log clog x
                                        $$

                                        So, not only $c^{log x}$ and $x^{log c}$ grow at the same rate: they're equal, whatever base of logarithms you use.



                                        For the second part, note that $x=e^{ln x}=b^{log_bx}$ by definition. Then
                                        $$
                                        ln x=log_bxln b
                                        $$

                                        Therefore, for $xne1$,
                                        $$
                                        frac{ln x}{log_bx}=ln b
                                        $$







                                        share|cite|improve this answer












                                        share|cite|improve this answer



                                        share|cite|improve this answer










                                        answered Jan 27 at 15:38









                                        egregegreg

                                        182k1485204




                                        182k1485204























                                            1












                                            $begingroup$

                                            $log_a b$ = $log_c b over log_c a$ is a general rule.
                                            Thus $log_c a = {log_c b over log_a b}$ and your expression is the case of this when $a=10, b=x, c=e$.






                                            share|cite|improve this answer











                                            $endgroup$













                                            • $begingroup$
                                              Ummm.... by your correct rule we have $$ln(10)=frac{log_x(10)}{log_x(e)}$$ which isn't what we're trying to prove.
                                              $endgroup$
                                              – Rhys Hughes
                                              Jan 27 at 6:54








                                            • 1




                                              $begingroup$
                                              Thanks. I edited to correct
                                              $endgroup$
                                              – J. W. Tanner
                                              Jan 27 at 7:10
















                                            1












                                            $begingroup$

                                            $log_a b$ = $log_c b over log_c a$ is a general rule.
                                            Thus $log_c a = {log_c b over log_a b}$ and your expression is the case of this when $a=10, b=x, c=e$.






                                            share|cite|improve this answer











                                            $endgroup$













                                            • $begingroup$
                                              Ummm.... by your correct rule we have $$ln(10)=frac{log_x(10)}{log_x(e)}$$ which isn't what we're trying to prove.
                                              $endgroup$
                                              – Rhys Hughes
                                              Jan 27 at 6:54








                                            • 1




                                              $begingroup$
                                              Thanks. I edited to correct
                                              $endgroup$
                                              – J. W. Tanner
                                              Jan 27 at 7:10














                                            1












                                            1








                                            1





                                            $begingroup$

                                            $log_a b$ = $log_c b over log_c a$ is a general rule.
                                            Thus $log_c a = {log_c b over log_a b}$ and your expression is the case of this when $a=10, b=x, c=e$.






                                            share|cite|improve this answer











                                            $endgroup$



                                            $log_a b$ = $log_c b over log_c a$ is a general rule.
                                            Thus $log_c a = {log_c b over log_a b}$ and your expression is the case of this when $a=10, b=x, c=e$.







                                            share|cite|improve this answer














                                            share|cite|improve this answer



                                            share|cite|improve this answer








                                            edited Jan 27 at 7:08

























                                            answered Jan 27 at 6:49









                                            J. W. TannerJ. W. Tanner

                                            2,1021117




                                            2,1021117












                                            • $begingroup$
                                              Ummm.... by your correct rule we have $$ln(10)=frac{log_x(10)}{log_x(e)}$$ which isn't what we're trying to prove.
                                              $endgroup$
                                              – Rhys Hughes
                                              Jan 27 at 6:54








                                            • 1




                                              $begingroup$
                                              Thanks. I edited to correct
                                              $endgroup$
                                              – J. W. Tanner
                                              Jan 27 at 7:10


















                                            • $begingroup$
                                              Ummm.... by your correct rule we have $$ln(10)=frac{log_x(10)}{log_x(e)}$$ which isn't what we're trying to prove.
                                              $endgroup$
                                              – Rhys Hughes
                                              Jan 27 at 6:54








                                            • 1




                                              $begingroup$
                                              Thanks. I edited to correct
                                              $endgroup$
                                              – J. W. Tanner
                                              Jan 27 at 7:10
















                                            $begingroup$
                                            Ummm.... by your correct rule we have $$ln(10)=frac{log_x(10)}{log_x(e)}$$ which isn't what we're trying to prove.
                                            $endgroup$
                                            – Rhys Hughes
                                            Jan 27 at 6:54






                                            $begingroup$
                                            Ummm.... by your correct rule we have $$ln(10)=frac{log_x(10)}{log_x(e)}$$ which isn't what we're trying to prove.
                                            $endgroup$
                                            – Rhys Hughes
                                            Jan 27 at 6:54






                                            1




                                            1




                                            $begingroup$
                                            Thanks. I edited to correct
                                            $endgroup$
                                            – J. W. Tanner
                                            Jan 27 at 7:10




                                            $begingroup$
                                            Thanks. I edited to correct
                                            $endgroup$
                                            – J. W. Tanner
                                            Jan 27 at 7:10











                                            1












                                            $begingroup$

                                            Because $e^{cx}=10^{x}$ for all $x$ and $c:=ln(10)approx 2.3025$






                                            share|cite|improve this answer











                                            $endgroup$


















                                              1












                                              $begingroup$

                                              Because $e^{cx}=10^{x}$ for all $x$ and $c:=ln(10)approx 2.3025$






                                              share|cite|improve this answer











                                              $endgroup$
















                                                1












                                                1








                                                1





                                                $begingroup$

                                                Because $e^{cx}=10^{x}$ for all $x$ and $c:=ln(10)approx 2.3025$






                                                share|cite|improve this answer











                                                $endgroup$



                                                Because $e^{cx}=10^{x}$ for all $x$ and $c:=ln(10)approx 2.3025$







                                                share|cite|improve this answer














                                                share|cite|improve this answer



                                                share|cite|improve this answer








                                                edited Jan 28 at 7:12

























                                                answered Jan 28 at 5:53









                                                BananachBananach

                                                3,84711429




                                                3,84711429






























                                                    draft saved

                                                    draft discarded




















































                                                    Thanks for contributing an answer to Mathematics Stack Exchange!


                                                    • Please be sure to answer the question. Provide details and share your research!

                                                    But avoid



                                                    • Asking for help, clarification, or responding to other answers.

                                                    • Making statements based on opinion; back them up with references or personal experience.


                                                    Use MathJax to format equations. MathJax reference.


                                                    To learn more, see our tips on writing great answers.




                                                    draft saved


                                                    draft discarded














                                                    StackExchange.ready(
                                                    function () {
                                                    StackExchange.openid.initPostLogin('.new-post-login', 'https%3a%2f%2fmath.stackexchange.com%2fquestions%2f3089210%2fwhy-is-the-ratio-of-lnx-and-logx-a-constant%23new-answer', 'question_page');
                                                    }
                                                    );

                                                    Post as a guest















                                                    Required, but never shown





















































                                                    Required, but never shown














                                                    Required, but never shown












                                                    Required, but never shown







                                                    Required, but never shown

































                                                    Required, but never shown














                                                    Required, but never shown












                                                    Required, but never shown







                                                    Required, but never shown







                                                    Popular posts from this blog

                                                    How do I know what Microsoft account the skydrive app is syncing to?

                                                    Grease: Live!

                                                    When does type information flow backwards in C++?